4
$\begingroup$

Let $1\leq p <\infty$ and let $p^{\prime}$ denote its conjugate exponent. Consider the following operator on Schwartz functions:

$$Tf(x)=\int_{0}^{\infty}t^{\frac{n}{2 p^{\prime}}-1}e^{-t} \int_{|x-y|^2\leq t}\frac{f(y)}{|x-y|^{\frac{n}{p^{\prime}}}}dy dt,\qquad x\in \mathbb{R}^{n}.$$

I have tried to prove that $T$ is bounded from $L^{p}$ to $L^{\infty}$ but failed so far.

Young's inequality for convolution is not useful with the $y$-integral as $|\cdot|^{\frac{n}{p^{\prime}}}$ is not in $L^{p^{\prime}}(B(t))$ with $B(t)$ the standard ball centered at the origin with radius $t>0$.

Hardy-Little-wood-Sobolev inequality is not useful for obtaining $L^{\infty}$ boundedness.

One could look at the Hardy-Littlewood maximal operator $\displaystyle Mf(x)=\sup_{r>0} \frac{1}{B(x,r)} \int_{B(x,r)}\frac{f(y)}{|x-y|^{\frac{n}{p^{\prime}}}}dy$ since
$$\frac{1}{t^{\frac{n}{2}}}\int_{|x-y|^2\leq t}\frac{f(y)}{|x-y|^{\frac{n}{p^{\prime}}}}dy\leq Mf(x).$$

The maximal operator is known to be bounded from $L^{p}$ to $L^{p}$ for all $1<p\leq \infty$ and from $L^1$ to weak $L^{1}$. I have no idea about the boundedness of $M$ from $L^p$ to $L^{\infty}$ when $p<\infty$.

Is it true that $$\|Tf\|_{L^{\infty}}\leq C \|f\|_{L^{p}}$$ for any $1\leq p<\infty$ or is there a counterexample ?

$\endgroup$
1
  • 1
    $\begingroup$ Yes indeed, no idea. $\endgroup$
    – user130023
    Oct 4, 2022 at 19:31

2 Answers 2

6
$\begingroup$

No this is not true. Take $n=1$, $p=2$ and $$f(y)=\frac{1}{\sqrt {|y|} (|\log|y||)^\alpha}\chi_{(-1,1)}(y)$$ with $\frac 12 < \alpha <1$. Then $f \in L^2(\mathbb R)$ but the innermost integral diverges at $x=0$ for every $t>0$.

EDIT The same counterexample can be done for general $n$ and $1<p<\infty$ (the case $p=1$ is in the answer by @Willie Wong). Take $$f(y)=\frac{1}{{|y|^{\frac np}} (|\log|y||)^\alpha}\chi_{B_r}(y)$$ with $r<1$ and $\frac 1p <\alpha <1$. Then $Tf(0)=\infty$ and, by Fatou, $\lim_{x \to 0}Tf(x)=\infty$.

$\endgroup$
5
  • 1
    $\begingroup$ +1 for the general case! I got confused by the question last night, because the OP mentioned maximal functions. But because the inner integration is with $|x-y|^2 \leq t$ (and not the other way around) the outer integral cannot regularize away any singularity at zero. $\endgroup$ Oct 5, 2022 at 13:58
  • $\begingroup$ @Giorgio Metafune It is easy to see that $\int_{|y|<r<1}\frac{dy}{|y|(|\log|y||)^\alpha}=\infty$, but why is $\liminf_{|x|\rightarrow 0} \int_{|x-y|<t,\,|y|<r<1}\frac{dy}{{|x-y|^{\frac{n}{p^{\prime}}}|y|^{\frac np}} (|\log|y||)^\alpha}=\infty$ ? $\endgroup$
    – Medo
    Oct 5, 2022 at 16:57
  • $\begingroup$ Because it is bigger of the integral with liminf inside. $\endgroup$ Oct 5, 2022 at 17:02
  • $\begingroup$ So we are using Fatou's lemma twice actually ? $\endgroup$
    – Medo
    Oct 5, 2022 at 17:07
  • 1
    $\begingroup$ Yes, one for the integral in $y$, the other in $t$. $\endgroup$ Oct 5, 2022 at 17:10
7
$\begingroup$

The case $p = 1$ asks if $$\int_0^\infty t^{-1} e^{-t} \int_{|x-y|^2\leq t} f(y) dy dt \leq 1 $$ for every $f$ with $\int |f| = 1$. Just take $|f_n|$ a family approximating the identity with support on $B(0,1/n)$, then you see that the corresponding integral blows up.

For $n < 2p'$ (this includes the case that Giorgio Metafune considered), let $f$ be a smooth function that is concentrated on the dyadic annulus of size $2^{-k}$ with height 1. Then its $L^p$ norm is of size $2^{-kn/p}$. For $\sqrt{t} \approx 2^{-k}$, the inner integral evaluates to the same value. So the full integral is bounded below by $$ 2^{-kn/p} \cdot 2^{-k(n/p' - 2)} $$ for $k > 0$. So we see that when $n < 2p'$ taking $k\nearrow \infty$ you also get counterexamples to the boundedness.

I think with some work a similar counterexample can be made for $n = 2p'$. But I am not sure about what happens when $n > 2p'$.

$\endgroup$
0

Your Answer

By clicking “Post Your Answer”, you agree to our terms of service and acknowledge you have read our privacy policy.

Not the answer you're looking for? Browse other questions tagged or ask your own question.